Download as pdf or txt
Download as pdf or txt
You are on page 1of 2

MATH 172: MIDTERM EXAMINATION: SOLUTIONS

April 26, 2011

1. (20 points)
(a) Let X be a set, R be a ring, and let µ be a measure on R. Let µ∗ denote
the corresponding outer measure. If µ∗ (A) = 0 then prove that µ∗ (B) = µ∗ (A ∪ B)
for any subset B of X.
Solution. Since B ⊂ A ∪ B we have µ∗ (B) ≤ µ∗ (A ∪ B). By subadditivity of outer
measure we have µ∗ (A ∪ B) ≤ µ∗ (A) + µ∗ (B) ≤ µ∗ (B). Hence µ∗ (B) = µ∗ (A ∪ B).
(b) Let R denote the smallest σ-field that includes all the Borel sets of R and all
sets of outer measure zero. Is R the same as the collection of Lebesgue measurable
sets?
Solution. Yes. Recall that every Lebesgue measurable set A can be written as
B ∪ E where B is a Borel set, and E has measure zero. Thus every Lebesgue
measurable set is contained in R. Since R was supposed to be the smallest σ-filed
containing all the Borel sets, and sets of measure zero, it must coincide with the
collection of Lebesgue measurable sets.
(c) Let A ⊂ R be a Lebesgue measurable set with positive measure. Is it true
that A contains an open interval?
Solution. No. Consider the irrational numbers in (0, 1) which is a set of measure
1 containing no open interval.
2. (30 points) Let E be a set of real numbers and let E 2 = {x2 : x ∈ E}.
(a). If E is bounded and has outer measure zero then show that E 2 also has
outer measure zero. Generalize to the case when E is not necessarily bounded, but
still has outer measure zero.
Solution. Suppose that E is bounded so that all elements of E are bounded by
some number N say. Since E has outer measure zero, given any  > P0 we may
cover E by a countable union of intervals Ij with Ij ⊂ [−N, N ] and j |Ij | < .
Then E 2 is covered by the union of Ij2 , and note that |Ij |2 ≤ 2N |Ij | (why?). Hence
µ∗ (E 2 ) ≤
P 2 2
j |Ij | ≤ 2N  and since  was arbitrary we find that E has outer
measure zero.
For the unbounded case, write EN = E ∩ [−N, N ] so that E = ∪∞ N =1 EN and
E = ∪∞
2
E
N =1 N
2
. From the above we know that the outer measure of EN 2
is zero,
and so by countable subadditivity of outer measure we find that the outer measure
of E is also zero.
(b). If E is a Borel set then so also is E 2 .
Solution. Let C denote the collection of all subsets E of R for which E 2 is a Borel
set. Note that C is a σ-field (why? – be careful about E = A − B). If E is an open
set in (0, ∞) then E 2 is an open set (why?). If E is an open set in (−∞, 0) then

Typeset by AMS-TEX
1
2 APRIL 26, 2011

E 2 is an open set. Thus C contains all open subsets of (0, ∞) and (−∞, 0), and
hence contains all open subsets in R (why?). Since the Borel sets form the smallest
σ-field containing all open sets in R, we conclude that C contains all the Borel sets.
2
√ way to say this is that on (0, ∞) the function x →√x has a continuous
Another
inverse x, and on (−∞, 0) there is a continuous inverse − x. Therefore Borel
sets in (0, ∞) and (−∞, 0) get mapped to Borel sets ...
(c). If E is Lebesgue measurable then so also is E 2 .
Solution. If E is Lebesgue measurable then we may write it as A ∪ B where A
has measure zero and B is Borel. Then E 2 = A2 ∪ B 2 and B 2 is Borel, and A2 has
measure zero. Thus E 2 is Lebesgue measurable.
P∞
3. (20 points). Write a number x ∈ [0, 1) as x = n=1 an /3n where an = 0, 1, or
2 and if there are two such ternary expansions pick the one ending in zeros. Let
f : [0, 1) → N ∪ {∞} be defined as f (x) = N where N is the least j with aj = 1,
and setting N = ∞ if no such j exists. Prove that f is measurable.
Solution. It suffices to show that for every c, the set {x : f (x) < c} is measurable.
But for each N < ∞, the set {x : f (x) = N } is a union of 2N −1 intervals each of
length 1/3N , and so it is measurable.
4. (30 points) Let A ⊂ [0, 1] be the set of all α for which there exist infinitely many
integers a and q with q ≥ 1 and
a 1
α − ≤ 3 .

q q

Prove that A is measurable and determine its measure.


Solution. Let Aq = ∪qa=0 [a/q−1/q 3 , a/q+1/q 3 ]. Then A consists of those elements
in [0, 1] lying in infinitely many Aq . That is
 
A = [0, 1] ∩ ∩∞
k=1 ∪∞
q=k Aq ,

and so A is a Borel, andPqhence measurable, set. P∞ P∞


Note that µ(Aq ) ≤ a=0 2/q 3 = 2(q + 1)/q 3 . Since q=1 µ(Aq ) ≤ q=1 (2/q 2 +
2/q 3 ) < ∞, by the first Borel-Cantelli Lemma we see that A has measure zero.

You might also like